• 沒有找到結果。

1061ᵬᵬᵬ03-04ஹஹஹ06-10IIIʟʟʟὃὃὃYYYȤȤȤᑖᑖᑖ 1. (20 points) Compute each of the following limits if it exists or explain why it doesn’t exist. (a) (5 points) lim

N/A
N/A
Protected

Academic year: 2022

Share "1061ᵬᵬᵬ03-04ஹஹஹ06-10IIIʟʟʟὃὃὃYYYȤȤȤᑖᑖᑖ 1. (20 points) Compute each of the following limits if it exists or explain why it doesn’t exist. (a) (5 points) lim"

Copied!
14
0
0

加載中.... (立即查看全文)

全文

(1)

1061微微微甲甲甲03-04、、、06-10班班班期期期中中中考考考解解解答答答和和和評評評分分分標標標準準準

1. (20 points) Compute each of the following limits if it exists or explain why it doesn’t exist.

(a) (5 points) lim

x→0sin(x12) sin x.

(b) (5 points) lim

x→0 tan x

1−cos 3x. (c) (5 points) lim

x→0(cos x)x22 . (d) (5 points) lim

x→+∞

√x+√ x−√

x−√ x.

Solution:

(a) Method 1

(2 points) −1 ≤ sinx12 ≤ 1 ⇒ 0 ≤ ∣ sinx12∣ ≤ 1 ⇒ 0 ≤ ∣ sin x sinx12∣ ≤ ∣ sin x∣

(2 points) lim

x→0sin x= 0 ⇒ limx→0∣ sin x∣ = 0 = limx→00.

(1 point) By Squeeze Theorem, lim

x→0∣ sin x sinx12∣ = 0 ⇒ limx→0sin x sinx12 = 0.

Method 2 To find the limit as x approaching 0, only to consider x∈ (−π, π).

(1 point) When x> 0, −1 ≤ sinx12 ≤ 1 ⇒ − sin x ≤ sin x sinx12 ≤ sin x.

(1 point) lim

x→0+− sin x = 0 = limx→0+sin x.

(1 point) When x< 0, −1 ≤ sinx12 ≤ 1 ⇒ sin x ≤ sin x sinx12 ≤ − sin x.

(1 point) lim

x→0sin x= 0 = limx→0− sin x.

(1 point) By Squeeze Theorem, lim

x→0+sin x sinx12 = 0 = limx→0sin x sinx12. Hence, lim

x→0sin x sinx12 = 0.

(b) (2 points) tan x

1−cos 3x = tan xx

√ x

1−cos 3x = (cos x1 x sin x)(3∣x∣x

(3x)2

1−cos(3x)) = 13 1 cos x

sin x x

x

∣x∣

(3x)2

1−cos(3x)

for x≠ 0.

(1 point) lim

x→0+ tan x

1−cos 3x = limx→0+(13cos x1 sin xx ∣x∣x(3x)2

1−cos(3x)) = 13⋅ 1 ⋅ 1 ⋅ 1 ⋅√

2=32. (1 point) lim

x→0 tan x

1−cos 3x = limx→0(13 1 cos x

sin x x

x

∣x∣

(3x)2

1−cos(3x)) = 13⋅ 1 ⋅ 1 ⋅ (−1) ⋅√

2= −32. (1 point) lim

x→0+ tan x

1−cos 3x = 32 ≠ −32 = lim

x→0 tan x

1−cos 3x ⇒ limx→01−cos 3xtan x doesn’t exist.

(c) Observe that

limx→0(cos(x))2/x2 = limx→0e2 ln(cos(x))/x2 = ex→0lim2 ln(cos(x))/x2

(1 pt).

Now since

limx→0

2 ln(cos(x)) x2

L= limx→02× − sin(x)/ cos(x)

2x (2 pts)

= limx→0− sin(x)

x × 1

cos(x)

= −1 (1 pt) × 1

= −1 (1 pt), we conclude that lim

x→0(cos(x))2/x2 = ex→0lim2 ln(cos(x))/x2

= e−1.

(2)

(d)

x→+∞lim

√x+√ x−√

x−√

x = limx→+∞(x +√

x) − (x −√ x)

√x+√ x+√

x−√ x

(2 pts)

= limx→+∞ 2√

√ x x+√

x+√ x−√

x = limx→+∞ 2

√ 1+√

x x +

√ 1−√

x x

= limx→+∞ 2

√ 1+ 1

√x+

√ 1− 1

√x

(2 pts) = 2

√1+ 0 +√ 1− 0

= 2

1+ 1 = 1 (1 pt)

(3)

2. (15 points) Differentiate the following functions.

(a) (5 points) f(x) =1+cos xsin x . (b) (5 points) f(x) = log2

√x+ tan−1(x3).

(c) (5 points) f(x) = xcos x.

Solution:

(a) (Method 1) Use quotient rule

f(x) =

³¹¹¹¹¹¹¹¹¹¹¹¹¹¹¹¹¹¹¹¹¹¹¹¹¹¹¹¹¹¹¹¹¹¹¹¹¹¹¹·¹¹¹¹¹¹¹¹¹¹¹¹¹¹¹¹¹¹¹¹¹¹¹¹¹¹¹¹¹¹¹¹¹¹¹¹¹¹¹µ1pt cos x(cos x + 1) −

³¹¹¹¹¹¹¹¹¹¹¹¹¹¹¹¹¹¹¹¹¹¹¹¹¹¹¹¹¹¹·¹¹¹¹¹¹¹¹¹¹¹¹¹¹¹¹¹¹¹¹¹¹¹¹¹¹¹¹¹¹¹µ1pt sin x(− sin x) (1 + cos x)2

´¹¹¹¹¹¹¹¹¹¹¹¹¹¹¹¹¹¹¹¹¹¹¹¹¸¹¹¹¹¹¹¹¹¹¹¹¹¹¹¹¹¹¹¹¹¹¹¹¹¶

1pt

= 1+ cos x

(1 + cos x)2 = 1 1+ cos x

(All correct +2pts.)

(Method 2) Use product rule

f(x) = cos x

1+ cos x + (sin x) ⋅ (− (1+ cos x) (1 + cos x)2) =

³¹¹¹¹¹¹¹¹¹¹¹¹¹¹·¹¹¹¹¹¹¹¹¹¹¹¹¹¹¹µcos x1pt 1+ cos x +

³¹¹¹¹¹¹¹¹¹¹¹¹¹¹¹¹¹¹¹¹¹¹¹¹¹¹¹¹¹¹·¹¹¹¹¹¹¹¹¹¹¹¹¹¹¹¹¹¹¹¹¹¹¹¹¹¹¹¹¹¹µ2pts sin2x (1 + cos x)2) (All correct +2pts.)

(b) (Method 1) Simplify f(x) as

f(x) =

­1pt ln x

2 ln 2 + tan−1(x3) Then

f(x) =

³¹¹¹¹¹¹¹¹¹¹¹¹¹¹·¹¹¹¹¹¹¹¹¹¹¹¹¹¹¹µ1pt 1 (2 ln 2)x +

«1pt 3x2

³¹¹¹¹·¹¹¹¹µ1pt 1 1+ x6 (All correct +1pt.)

(Method 2) Differentiate f(x) directly

f(x) =

³¹¹¹¹¹¹¹¹¹¹¹¹¹¹¹¹·¹¹¹¹¹¹¹¹¹¹¹¹¹¹¹¹µ1pt 1 (ln 2)√

x⋅

¬1pt 1 2√

x+

«1pt 3x2

³¹¹¹¹·¹¹¹¹µ1pt 1 1+ x6 (All correct +1pt.)

(c) (Method 1) Write f(x) as

³¹¹¹¹¹¹¹¹¹¹¹¹¹¹¹¹¹¹¹¹¹¹¹¹¹¹¹¹¹¹¹¹¹¹¹¹¹·¹¹¹¹¹¹¹¹¹¹¹¹¹¹¹¹¹¹¹¹¹¹¹¹¹¹¹¹¹¹¹¹¹¹¹¹¹µ1pt f(x) = ecos x ln x Then differentiate f(x)

f(x) =

³¹¹¹¹¹¹¹¹¹¹¹¹¹¹¹¹¹¹¹¹¹¹¹¹¹¹¹¹¹¹¹¹¹¹¹¹¹¹¹¹¹¹¹¹¹¹¹¹¹¹¹¹¹·¹¹¹¹¹¹¹¹¹¹¹¹¹¹¹¹¹¹¹¹¹¹¹¹¹¹¹¹¹¹¹¹¹¹¹¹¹¹¹¹¹¹¹¹¹¹¹¹¹¹¹¹¹µ1pt

ecos x ln x(cos x ln x)= ecos x ln x

³¹¹¹¹¹¹¹¹¹¹¹¹¹¹¹¹¹¹¹¹¹¹¹¹¹¹¹¹¹¹¹¹¹¹¹¹¹¹¹¹¹¹¹¹¹¹¹¹¹¹¹¹¹¹¹¹¹·¹¹¹¹¹¹¹¹¹¹¹¹¹¹¹¹¹¹¹¹¹¹¹¹¹¹¹¹¹¹¹¹¹¹¹¹¹¹¹¹¹¹¹¹¹¹¹¹¹¹¹¹¹¹¹¹¹¹µ1pt (− sin x ln x +cos x

x )

(4)

(All correct +2pts.)

(Method 2) Use logarithmic differentiation. Write f(x) as

³¹¹¹¹¹¹¹¹¹¹¹¹¹¹¹¹¹¹¹¹¹¹¹¹¹¹¹¹¹¹¹¹¹¹¹¹¹¹¹¹¹¹¹¹¹¹¹¹¹¹¹·¹¹¹¹¹¹¹¹¹¹¹¹¹¹¹¹¹¹¹¹¹¹¹¹¹¹¹¹¹¹¹¹¹¹¹¹¹¹¹¹¹¹¹¹¹¹¹¹¹¹¹µ1pt ln f(x) = cos x ln x Differentiate

­1pt f(x)

f(x) =

³¹¹¹¹¹¹¹¹¹¹¹¹¹¹¹¹¹¹¹¹¹¹¹¹¹¹¹¹¹¹¹¹¹¹¹¹¹¹¹¹¹¹¹¹¹¹¹¹¹¹¹¹¹¹¹¹¹·¹¹¹¹¹¹¹¹¹¹¹¹¹¹¹¹¹¹¹¹¹¹¹¹¹¹¹¹¹¹¹¹¹¹¹¹¹¹¹¹¹¹¹¹¹¹¹¹¹¹¹¹¹¹¹¹¹¹µ1pt (− sin x ln x +cos x

x ) Thus,

f(x) = xcos x(− sin x ln x +cos x x ) (All correct +2pts.)

Remark 計算錯誤至少扣2分,答案正確但沒有計算過程或說明扣1分,關鍵過程大致正確但抄寫錯

誤扣1分。

(5)

3. (12 points) Let f(x) =⎧⎪⎪⎪

⎨⎪⎪⎪⎩

x43 cos(1

x) , for x ≠ 0

0, for x= 0.

(a) (3 points) Is f(x) continuous at x = 0?

(b) (6 points) Compute f(x) for x ≠ 0 and f(0).

(c) (3 points) Is f(x) continuous at x = 0?

Solution:

(a) Because the cosine of any number lies between −1 and 1, we can write.

−1 ≤ cos (1 x) ≤ 1

Any inequality remains true when multiplied by a positive number. We know that x43 ≥ 0 for all x and so, multiplying each side of the inequalities by x43, we get

−x43 ≤ x43 cos(1 x) ≤ x43 We know that

limx→0x43 = limx→0−x43 = 0 By Squeeze Theorem, we obtain

limx→0x43 cos(1 x) = 0 Therefore, f(x) is continuous at x = 0.

(b) By definition of differential and pinching theorem,

f(0) = limx→0x43 cos(1x) − f(0)

x− 0 = limx→0x43 cos(1x)

x = limx→0x13cos(1 x) = 0

On the other hand, we consider x /= 0. By the Product Rule, f(x) = (x43)cos(x1) + x43{cos (1x)}. Now by Chain Rule, {cos (1x)}= sin (1x) ⋅ x12. Therefore, we obtain

f(x) =4

3x13cos(1

x) + x−23 sin(1 x) (c) Because lim sup

x→0

f(x) = ∞ and lim infx→0 f(x) = −∞, limx→0f(x) does not exist. Therefore, we can deduce that f(x) is not continuous at x = 0.

[Remark]

In question (b), suppose you know the definition of f(0) = limx→0f (x)−f (0)

x−0 . You can get 2 points.

Suppose you write f(0) = limx→0f(x). You can not get any point in (b).

(6)

4. (10 points) The figure shows a lamp located 4 units to the right of the y−axis and a shadow created by the elliptical region x2+5y2≤ 6. If the point (−6, 0) is on the edge of the shadow, how far above the x−axis is the lamp located?

? x y

0 3 _5

≈+4¥=5

x

2

+ 5y

2

= 6

. 6 4

Solution:

1. (Method 1)

By implicit differentiation, we have 2x+ 10yy= 0, or, y= −5yx (5%)

Suppose the point of tangency is (xo, yo), then the tangent line is given by y = −5yxoo(x − xo) + yo

Plug in (−6, 0), we have 0 = −5yxoo(−6 − xo) + yo, or, x2o+ 5yo2= −6xo. Also, x2o+ 5y2o = 6, so xo= −1, yo= 1(3%)

Then the tangent line is y =15x+65, so y∣

x=4= 2(2%) 2. (Method 2)

Suppose the lamp is located at (4, h). Then the tangent line is given by y = 10h(x+6)(4%) Since it’s tangent to the ellipse, the equation

⎧⎪⎪⎨⎪⎪

y= 10h(x + 6) x2+ 5y2= 6

should have only one zero(repeated roots), or equivalently, the discriminant of x2 + 5(10h(x + 6))2= 6 sholud be zero.(4%)

Thus, 36h4− (20 + h2)(36h2− 120) = 0, we have h = 2(2%) 3. (Method 3) Suppose the point of tangency is(xo,

√6−x2o

5 ), and the lamp is located at (4, h) Then we have

6−x2o 5 −0

xo−(−6) = 4−(−6)h−0 , or h= 2

√ 30−5x2o xo+6 (4%) We can find h by using the condition dhdx = 0(why?)(4%) Thus, 0=

−10xo

30−5x2o

⋅(xo+6)−2√ 30−5x2o⋅1

(xo+6)2 = −10x(xo2o+6)−60x2o−60+10x2o 30−5x2o

Hence, xo= −1, yo= 1, and h = 2(2%)

(7)

5. (10 points)

(a) (3 points) Find the linearization of f(x) = sin x at π6.

(b) (7 points) Explain why f satisfies the hypotheses of the Mean Value Theorem on [π6,π2]. Use the theorem to prove that

sin x< 1 2+

√3 2 (x −π

6) for x ∈ (π 6,π

2]

Solution:

5. (10 points)

(a) (3 points) Find the linearization of ( ) sin at 6

(b) (7 points) Explain why satisfies the hypotheses of the Mean Value Theorem on , . Use the theorem to prove that

6 2

f x x

f

 

 

 

 

1 3

sin ( ) for ( , ]

2 2 6 6 2

x  x

x

 

0 0 0 0

6

Solution:

(a)

The linearization of ( ) is

( ) ( ) ( )( ), 6

1 3

Note that ( ) and ( ) (sin ) cos

6 2 6 6 2

Therefore, the linearization o

(2

f ( ) at i

pt

6 s

f

s)

x

f x

L x f x f x x x x

f f x

f x

  

    

 

   

1 3

( ( ) ) ( )

2 2 6

(b)

( ) sin is differentiable on .

So it is continuous on the close interval [ , ], and is dif (1 pt for cor

ferentiable on the open 6 2

interval

rect equation)

( , ) sa 6 2

L x

f

x

f x x

 

 

  

 R

tisfying the hypotheses of the Mean Value Theorem on [ , ].

6 2

If is in the interval ( , ], we can still apply MVT on [ , ] to obtain a in

6 2 6

( , ) such that:

6

(2 pts for required hypoth

es s) e

x x c

x

 

  

sin sin ( ) cos 6

6

Since cos is strictly decreasing on ( , ) (

(2 pts for applying MVT correctly)

(2

(cos ) 0), we get the inequality 6 2

cos 3 for c ( , ) pts

2 6 2

x

f c c

x

x x

c

 

 

   

 

 

 )

(8)

Then

sin sin 6 3

( ) cos for ( , ]

2 6 2

6 Therefore,

1 3

sin ( ) for ( , ]

(

2 2 6 6 2

1 pt for deducing the inequality)

x

f c c x

x

x x x

  

  

     

   

: using the racetrack principle Set

1 3

( ) sin and ( ) ( )

2 2 6

( ) 3 2

( ) cos and ( ) 3

6 2

Since (cos ) 0 on ( , ], cos is decreasing on ( , ]

6 2 6 2

Thus

( ) ( ) f

f x x L x x

L x

f x x f

x x

f x L x

   

   

 

   

 

   Method 2

or ( , ] 6 2 Note that

( ) ( ) 1

6 6 2

By the racetrack principle, ( ) ( ) for ( , ]

6 2 Therefore, we get the inequality

1 3

sin ( ) for ( , ]

2 2 6 6 2

x

f L

f x L x x

x x x

 

 

 

  

 

 

   

(9)

6. (10 points) In the engine, a 7-inch connecting rod is fastened to a crank of radius 3 inches. The crankshaft(機軸) rotates counterclockwise at a constant rate of 100 revolutions per minute. Find the velocity of the piston(活塞) when θ = π3. (Reminder: the angular velocity of a circular motion at a constant speed of 1 revolution per minute is 2π rad/min.)

Solution:

First, we know dt = 100 ⋅ 2π = 200π (1 point) (Idea I)

∵72= 32+ x2− 2 ⋅ 3 ⋅ x cos θ (3 points)

∴0 = 2xdxdt − 6(dxdt cos θ− x sin θdt)

⇒ (6 cos θ − 2x)dxdt = 6x sin θdtdxdt = 6 cos θ−2x6x sin θ dt (3 points) When θ= π3, then x= 8. ⇒ dxdt = −4800133π. (2 points) (Idea II)

∵72= 32+ x2− 2 ⋅ 3 ⋅ x cos θ ⇒ x2− 6 cos θ − 40 = 0 (3 points)

∴x = 3 cos θ +√

9 cos2θ+ 40 (for x > 0) ⇒ dxdt = −3 sin θdt +12⋅ −18 cos θ sin θ√

9 cos2θ+40 ⋅dt (3 points) when θ= π3, we have dxdt = [−3 ⋅ (23) −1218⋅12

3

2 9

4+40] ⋅ 200π = −4800133π (2 points) (Idea III)

∵ cos θ = 32+x2⋅3⋅x2−72(⇒ x = 3 cos θ +√

9 cos2θ+ 40 (for x > 0)) (3 points)

∴ − sin θdt = 2x⋅6xdxdt−6(x36x22−40)dxdt ⇒ − sin θdt =x26x+402 dx

dt (3 points) When θ= π3, then x= 8. ⇒ dxdt = − sin θdt ⋅x6x2+402

x=8,θ=π3 = −4800133π. (2 points) (Idea IV)

∵x = 3 cos θ +√

49− (3 sin θ)2(= 3 cos θ +√

40+ 9(1 − sin2θ) = 3 cos θ +√

9 cos2θ+ 40) (3 points)

dxdt = −3 sin θdt +12⋅−18 sin θ cos θ√

49−9 sin2θ ⋅dt (3 points) when θ= π3, we have dxdt = [−3 ⋅ (23) −12

18⋅

3 21

2

49−274 ] ⋅ 200π = −4800133π (2 points) (Idea V)

7 sin φ= 3 sin θ ⇒ 7 cos φdt = 3 cos θdt

dt =7 cos φ3 cos θ

dt = 49−49 sin3 cos θ 2φ dθ

dt = 49−9 sin3 cos θ2θ dθ

dt (2 points)

∵x = 3 cos θ + 7 cos φ (= 3 cos θ + 7√

1−499 sin2θ= 3 cos θ + 7√

49− 9 sin2θ) (1 point)

dxdt = −3 sin θdt − 7 sin φdt = −3 sin θdt − 3 sin θ (49−9 sin3 cos θ2θ dθ

dt) (3 points) when θ= π3, we have dxdt = [−3 ⋅ (23) −1218⋅

3 21

2

49−274 ] ⋅ 200π = −4800133π (2 points) (Idea VI)

7 sin φ= 3 sin θ ⇒ 7 cos φdt = 3 cos θdt, when θ= π3 ⇒ sin φ = 3143, cos φ=1314

dt =7 cos φ3 cos θdt = 49−49 sin3 cos θ 2φ dθ

dt = 49−9 sin3 cos θ2θ dθ

dt (2 points)

∵sin(π−φ−θ)x = sin θ7 ⇒ x = 7sin(π−φ−θ)sin θ = 7sin(θ+φ)sin θ (1 point)

(10)

dxdt = 7cos(θ+φ)⋅(dt+dt)sin θ−cos θ sin(θ+φ)dt

sin2θ (3 points)

When θ= π3, we have dxdt = 7[(

1 2)(13

14)−(

3 2 )(3

3 14 )]⋅[

3

2 49− 274

+1]

3 212[(

3

2 )(1314)+(1

2)(3

3 14 )]

3

4 ⋅ 200π

dxdt = 7 [(284) (1613)2316563] ⋅8003 π= 7 ⋅−9⋅16⋅8003⋅56⋅13 = −4800133π (2 points) That is the velocity of the piston is 4800

√ 3

13 π inch/min. with the direction to the left. (1 point)

(11)

7. (10 points) A right circular cone is inscribed in a larger right circular cone so that its vertex is at the center of the base of the larger one. Denote the height of the large cone by H and the height of the small one by h. When the large cone is fixed, find h that maximizes the volume of the small cone and find out this maximum volume in terms of the volume of the large cone. (Hint: The volume of a right circular cone with height h and base radius r is 13πr2h.)

H

h

Solution:

We denote the radii of the bases of the larger and smaller cones as R and r, respectively. Then we have the relation

H− h

H = r

R. This gives us

r= R

H(H − h).

(2 points)

Hence the volume of the small cone is

Vsmall= 1 3πr2h

= 1

3πh⋅ R2

H2(H − h)2

for 0≤ h ≤ H

(2 points). In order to compute the maximum of V

small, we compute dVsmall

dh = πR2

3H2(H − h)(H − 3h).

(2 points)

By setting dVsmalldh = 0 we have h = H or h = H/3

(2 points). Since

V(0) = 0, V(H) = 0, V(H

3) = 4

81πR2H> 0, we know that when h= H/3, the maximum of Vsmall is

Vsmall= 4

81πR2H= 4

27Vlarge,

(2 points)

where Vlarge= 13πR2H is the fixed volume of the larger cone.

註1: h= H3 and Vsmall =274Vlarge should both be answered. Not answering both of the two will cost you 2 points.

註2: Assuming H = 2R will cost you 2 points.

(12)

8. (18 points) Let f(x) = ln ∣x∣x , x≠ 0. Answer the following questions by filling each blank below and give your reasons (including computations). Put None in the blank if the item asked does not exist.

(a) (3 points) Find all asymptote(s) of the curve y= f(x).

Vertical asymptote(s): .

Horizontal asymptote(s): .

Slant saymptote(s): .

(b) (4 points) f(x) is increasing on the interval(s) .

f(x) is decreasing on the interval(s) .

(c) (2 points) Find all local extreme values of f(x).

Local maximum point(s): (x, f(x)) = .

Local minimum point(s): (x, f(x)) = .

(d) (4 points) f(x) is concave upward on the interval(s) .

f(x) is concave downward on the interval(s) .

(e) (2 points) List the inflection point(s) of the curve y= f(x) ∶ (x, f(x)) = . (f) (3 points) Sketch the graph of f , and indicate all asymptotes, extreme values, and inflection

points.

Solution:

(a) 1. Vertical asymptote:

Answer : x= 0 (y-axis). Correctness : 0.5 point ; Explanation : 0.5 point.

Solution:

x> 0: limx→0+f(x) = limx→0+(ln(x)x ) = −∞0 = −∞

( means f(x) → −∞ when x → 0+).

x< 0: limx→0f(x) = limx→0(−ln(−x)−x ) = +∞0 = +∞

( means f(x) → +∞ when x → 0).

Note: We can’t use L’Hospital’s Rule to solve this question.

2. Horizontal asymptote:

Answer : y= 0 (x-axis). Correctness : 0.5 point ; Explanation : 0.5 point.

Solution:

x> 0: limx→+∞f(x) = limx→+∞(ln(x)x ) = limx→+∞(1x1) = 01 = 0 ( +∞+∞ type, use L’Hospital’s Rule.)

( means f(x) → 0 when x → +∞).

x< 0: limx→−∞f(x) = limx→−∞(−ln(−x)−x ) = limx→−∞(1x1) = 01 = 0

(13)

x> 0: limx→+∞f (x)x = limx→+∞(ln(x)x2 ) = limx→+∞(2⋅xx1 ) = +∞0 = 0 ( +∞+∞ type, use L’Hospital’s Rule.)

(This result is in contradiction, therefore a slant asymptote doesn’t exist).

x< 0: limx→−∞f (x)x = limx→−∞(ln(−x)x2 ) = limx→−∞(2⋅x1x ) = −∞0 = 0 ( +∞+∞ type, use L’Hospital’s Rule.)

(This result is in contradiction, therefore a slant asymptote doesn’t exist).

Solution2:

A slant asymptote of f(x) only occurs when x → ±∞, but limx→±∞f(x) = 0 from the above, therefore a slant asymptote doesn’t exist.

(b) Answer : increasing interval:(0, e) ∪ (−e, 0), decreasing interval:(e, +∞) ∪ (−∞, −e).

Correctness : 1 point per question;

Determine f(x) (1 point) + Illustrate f(x) > 0 and f(x) < 0 (1 point).

Solution:

x> 0: f(x) = ln(x)x , f(x) = 1−ln(x)x2 (from the Quotient Rule).

f(x) > 0: 1 − ln(x) > 0 ⇒ ln(x) < 1 ⇒ 0 < x < e (increasing interval).

f(x) < 0: 1 − ln(x) < 0 ⇒ ln(x) > 1 ⇒ x > e (decreasing interval).

x< 0: f(x) = −ln(−x)−x , f(x) = 1−ln(−x)x2 (from the Quotient Rule).

f(x) > 0: 1 − ln(−x) > 0 ⇒ ln(−x) < 1 ⇒ −e < x < 0 (increasing interval).

f(x) < 0: 1 − ln(−x) < 0 ⇒ ln(−x) > 1 ⇒ x < −e (decreasing interval).

(c) Answer : local maximum point:(e,1e), local minimum point:(−e,−1e ).

Correctness : 0.5 point per question; Explanation : 1 point.

Solution:

Local maximum: Because f(x) is increasing in (0, e) and decreasing in (e, +∞), local maximum point occurs at (e, f(e)) = (e,1e) (f(e) = 0).

Local minimum: Because f(x) is decreasing in (−∞, −e) and increasing in (−e, 0), local minimum point occurs at(−e, f(−e)) = (−e,−1e ) (f(−e) = 0).

(d) (4 points) f(x) is concave upward on the interval(s)

(−e

32

, 0 ) ⋃(e

32

, ∞)

. f(x) is concave downward on the interval(s)

(0, e

32

) ⋃(−∞,−e

32

)

.

⎧⎪⎪⎨⎪⎪

f′′(x) = x > 0, ⇒ −1xx2−(1−ln x)2x

x4 =x(−3+2 ln x) x4

f′′(x) = x < 0, ⇒ −1xx2−(1−ln(−x))2x

x4 =x(−3+2 ln(−x)) x4

(1)

f′′(x) > 0 ⇒ (concave upward)

⎧⎪⎪⎪⎪

⎨⎪⎪⎪⎪⎩

x(−3+2 ln x)

x4 > 0, x > 0 ⇒ −3 + 2 ln x > 0 ⇒ ln x > 32 ⇒ x > e32

choose x > e

32

x(−3+2 ln(−x))

x4 > 0, x < 0 ⇒ −3 + 2 ln(−x) < 0 ⇒ ln(−x) > 32 ⇒ x > −e32

choose − e

32

< x < 0

(2)

(14)

f′′(x) < 0 ⇒ (concave downward)

⎧⎪⎪⎪⎪

⎪⎪⎨⎪⎪⎪

⎪⎪⎪⎩

x(−3+2 ln x)

x4 > 0, x > 0 ⇒ −3 + 2 ln x > 0 ⇒ ln x > 32 ⇒ x > e32

choose 0 < x < e

32

x(−3+2 ln(−x))

x4 > 0, x < 0 ⇒ −3 + 2 ln(−x) < 0 ⇒ ln(−x) > 32 ⇒ x > −e32

choose x < −e

32

(3)

score: answer 0.5 point separately, right concept 2 points. ( notify: if the graph of f lies above all of its tangents on an interval I, then called concave upward. ex: f′′(x) > 0 ⇒ concave upward)

if ⋃ write ⋂ lose 0.5 point

(e) (2 points) List the inflection point(s) of the curve y= f(x) ∶ (x, f(x)) =

(e

32

,

3

2e32

), (−e

32

, −

3

2e32

)

f′′(x) = 0 ⇒ x = e32 and x= −e32

f′′(e32) = 32e−32 and f′′(−e32) = −32e−32

score: answer 0.5 points separately, right concept 1 points. ( notify f′′(x) = 0, x = −e32 and e−32 )

(f) (3 points) Plot the f(x) and indicate asymptote, local point and infection point.

參考文獻

相關文件

3m long is revolved about one of its leg to generate a right circular cone.. Find the radius, height, and volume of the cone of the greatest volume that can be made

To see that this is true without using calculus, note that G = 0 is the equation of a sphere centered at the origin and F = 0 is the equation of a right circular cone with vertex at

So we check derivative of f and g, the small one would correspond to graph (b) and the other to (c)(i.e.. Moreover, f is

Its radius of convergence

Rotating the line  = 2 about the -axis creates a (right) circular cone with vertex at the origin and axis the -axis.. Thus the collection of all such points  is a

(10 points) A right circular cone is inscribed in a larger right circular cone so that its vertex is at the center of the base of the larger one.. Denote the height of the large cone

The molal-freezing-point-depression constant (Kf) for ethanol is 1.99 °C/m. The density of the resulting solution is 0.974 g/mL.. 21) Which one of the following graphs shows the

The difference in heights of the liquid in the two sides of the manometer is 43.4 cm when the atmospheric pressure is 755 mm Hg.. 11) Based on molecular mass and dipole moment of